Invariancia de calibre y la forma de la acción de Rarita-Schwinger

en Weinberg vol. En la sección 5.9 (en particular, la página 251 y la discusión circundante), se explica que el operador de campo de menor dimensión para una partícula sin masa de espín-1 toma la forma de una intensidad de campo, F m v . Esto se debe a que un campo vectorial sin masa no se transforma adecuadamente bajo las transformaciones de Lorentz (Weinberg, Vol. I eq. 5.9.22) y requiere simetría/redundancia de calibre adicional para "modificar" los grados de libertad no físicos.

Una de las preguntas del libro de texto Modern QFT de Banks (problema 2.10) pide repetir este análisis general para los campos spin-3/2 y spin-2, para los cuales sabemos que los principales ejemplos son el campo de Rarita-Schwinger y el gravitón. Sin embargo, para spin-3/2, no entiendo por qué el campo Rarita-Schwinger se describe con una acción que se parece mucho a la acción de Dirac. ¿Cómo es posible que el campo de Rarita-Schwinger sin masa, que lleva un índice vectorial así como un índice spin-1/2 adicional, pueda evitar el mal comportamiento bajo ciertas transformaciones de Lorentz (que Weinberg llama S ( α , β ) ) que se exhibe en el vector sin masa?

Ingenuamente hubiera querido escribir un campo spin-3/2 como un

( 1 , 1 / 2 ) = ( 1 / 2 , 1 / 2 ) ( 1 / 2 , 0 )
representación del grupo de Lorentz para que el índice vectorial pueda tratarse independientemente del índice de espinor. ¿No debería continuar el argumento a favor de los fotones sin masa e implicar que debería tener algún tipo de 'intensidad de campo' para el campo de espín 3/2?

Sospecho que lo que me estoy perdiendo es algo trivial y tonto.

Respuestas (1)

Creo que estás un poco confundido con la representación de Lorentz. El campo se escribe con un índice de espinor de Dirac y de 4 vectores. Tan ingenuamente se transforma como [ ( 0 , 1 2 ) ( 1 2 , 0 ) ] ( 1 2 , 1 2 ) . Pero esto tiene dos representaciones irreducibles subducidas bajo el subgrupo de rotaciones espaciales, el spin- 3 2 campo de Rarita-Schwinger que desea, y un spin- 1 2 vas a proyectar. Esta es tu gran pista de que los índices de espinor y vector no son independientes. Deben ser "coordinados" para mantener sólo el giro de Rarita-Schwinger. 3 2 campo. Su representación bajo el grupo completo de Lorentz es ( 1 , 1 2 ) ( 1 2 , 1 ) .

Alternativamente, mirar la respuesta (es decir, la ecuación para el campo de Rarita-Schwinger) revela que los dos tipos de índices se refieren a los mismos términos y no son independientes entre sí.